SchoolWork-LaTeX/数理统计/平时作业/第十四周作业.tex
423A35C7 5906ac1efc 重构目录层次
0-课程笔记
1-平时作业
2-实验报告
3-期末大作业
2024-09-02 18:32:58 +08:00

189 lines
12 KiB
TeX
Raw Permalink Blame History

This file contains ambiguous Unicode characters

This file contains Unicode characters that might be confused with other characters. If you think that this is intentional, you can safely ignore this warning. Use the Escape button to reveal them.

\documentclass[全部作业]{subfiles}
\input{mysubpreamble}
\begin{document}
\setcounter{chapter}{7}
\setcounter{section}{2}
\begin{enumerate}
\questionandanswerSolution[14]{
在针织品漂白工艺过程中要考察温度对针织品断裂强力主要质量指标的影响。为了比较70℃与80℃的影响有无差别在这两个温度下分别重复做了8次试验得数据单位N如下
70℃时的强力\quad 20.5\quad 18.8\quad 19.8\quad 20.9\quad 21.5\quad 19.5\quad 21.0\quad 21.2,
80℃时的强力\quad 17.7\quad 20.3\quad 20.0\quad 18.8\quad 19.0\quad 20.1\quad 20.0\quad 19.l.
根据经验温度对针织品断裂强度的波动没有影响。问在70℃时的平均断裂强力与80℃时的平均断裂强力间是否有显著差别假定断裂强力服从正态分布取a= 0.05?
}{
使用$t$检验,检验的问题为
$$
H_0:\mu_1=\mu_2 \quad\mathrm{vs}\quad H_1:\mu_1\neq \mu_2
$$
根据样本计算得出$\bar{x}=20.4, \bar{y}=19.375, s_{u}=\sqrt{\frac{1}{8+8-2} (8\sigma^{2}_{x}+8\sigma^{2}_{y})}=0.9147599217$
\begin{center}
\includegraphics[width=0.3\linewidth]{imgs/2024-06-01-14-45-03.png}
\end{center}
$$
t=\frac{\bar{x}-\bar{y}}{s_u \sqrt{\frac{1}{8}+\frac{1}{8}}}=\frac{20.4-19.375}{0.9147599217 \sqrt{\frac{1}{8}+\frac{1}{8}}} \approx 2.2410, \quad W=\{ \left\vert t \right\vert \geqslant t_{0.975}(14) \}= \{ \left\vert t \right\vert \geqslant 2.1448 \}
$$
$t$在拒绝域内所以拒绝原假设所以在70℃时的平均断裂强力与80℃时的平均断裂强力间\boldkai{有显著差别}
再计算$p$值,$\displaystyle p=2(1-\Phi(\left\vert 2.2410 \right\vert ))=0.012513 < 0.05$,确实应拒绝原假设。
}
\questionandanswerSolution[15]{
一药厂生产一种新的止痛片,厂方希望验证服用新药片后至开始起作用的时间间隔较原有止痛片至少缩短一半,因此厂方提出需检验假设
$$
H_0:\mu_1=2\mu_2 \quad\mathrm{vs}\quad H_1:\mu_1>2\mu_2
$$
此处$\mu_1,\mu_2$分别是服用原有止痛片和服用新止痛片后至开始起作用的时间间隔的总体的均值。设两总体均为正态分布且方差分别为已知值$\sigma_1^{2},\sigma_2^{2}$,现分别在两总体中取一样本$x_1,x_2, \cdots ,x_n$$y_1,y_2, \cdots ,y_m$,设两个样本独立。试给出上述假设检验问题的检验统计量及拒绝域.
}{
使用$u$检验,检验统计量为 $\displaystyle u=\frac{\bar{x} - 2\bar{y}}{\sqrt{\frac{\sigma_1^{2}}{n}+\frac{4\sigma_2^{2}}{m}}}$,拒绝域为$\displaystyle W=\{ u\geqslant u_{1-\alpha} \}$
}
\questionandanswer[26]{
测得两批电子器件的样品的电阻(单位:$\Omega$)为
A批($x$):\qquad 0.140\quad 0.138\quad 0.143\quad 0.142\quad 0.144\quad 0.137;
B批($y$):\qquad 0.135\quad 0.140\quad 0.142\quad 0.136\quad 0.138\quad 0.140.
设这两批器材的电阻值分别服从分布$N(\mu_1, \sigma_1^{2}), N(\mu_2, \sigma_2^{2})$,且两样本独立。
}{
使用Excel计算如下
\begin{table}[H]
\tiny\centering
\begin{tabular}{ccccccccccc}
x & y & & & & & & & & & \\
0.14 & 0.135 & & & F-检验 双样本方差分析 & & & & t-检验: 双样本异方差假设 & & \\
0.138 & 0.14 & & & & & & & & & \\
0.143 & 0.142 & & & & x & y & & & x & y \\
0.142 & 0.136 & & & 平均 & 0.140667 & 0.1385 & & 平均 & 0.140667 & 0.1385 \\
0.144 & 0.138 & & & 方差 & 7.87E-06 & 7.1E-06 & & 方差 & 7.87E-06 & 7.1E-06\\
0.137 & 0.14 & & & 观测值 & 6 & 6 & & 观测值 & 6 & 6 \\
& & & & df & 5 & 5 & & 假设平均差 & 0 & \\
& & & & F & 1.107981 & & & df & 10 & \\
& & & & P(F<=f) 单尾 & 0.456576 & & & t Stat & 1.371845 & \\
& & & & F 单尾临界 & 5.050329 & & & P(T<=t) 单尾 & 0.100051 & \\
& & & & & & & & t 单尾临界 & 1.812461 & \\
& & & & & & & & P(T<=t) 双尾 & 0.200102 & \\
& & & & & & & & t 双尾临界 & 2.228139 & \\
& & & & & & & & & & \\
\end{tabular}
\end{table}
}
\begin{enumerate}
\questionandanswerSolution[]{
试检验两个总体的方差是否相等(取$\alpha=0.05$)。
}{
使用$F$检验,不能拒绝原假设,所以相等。
}
\questionandanswerSolution[]{
试检验两个总体的均值是否相等(取$\alpha=0.05$)。
}{
使用$t$检验,不能拒绝原假设,所以相等。
}
\end{enumerate}
\end{enumerate}
\section{其他分布参数的假设检验}
\begin{enumerate}
\questionandanswerSolution[2]{
某厂一种元件平均使用 寿命为1200 h偏低 现厂里进行技术革新革新后任选8个元件进行寿命试验测得寿命数据如下
$$
2686\quad 2001\quad 2082\quad 792\quad 1660\quad 4105\quad 1416\quad 2089
$$
假定元件寿命服从指数分布,取$\alpha=0.05$,问革新后元件的平均寿命是否有明显提高?
}{
使用$\chi^{2}$检验假设
$$
H_0: \theta \leqslant 1200 \quad\mathrm{vs}\quad H_1:\theta>1200
$$
$\chi^{2} = \frac{2\times 8 \bar{x}}{1200}\approx 28.0517$,拒绝域为$\{ \chi^{2}\geqslant \chi^{2}_{0.95}(2\times 8)\approx 26.2962 \}$,所以拒绝原假设,革新后元件的平均寿命\boldkai{有明显提高}
}
\questionandanswerSolution[3]{
有人称某地成年人中大学毕业生比率不低于30\%。为检验之随机调查该地15名成年人发现有3名大学毕业生$\alpha=0.05$,问该人看法是否成立?并给出检验的$p$值。
}{
样本的分布为$x\sim b(15, p')$,检验的假设为
$$
H_0: p'\geqslant 0.3 \quad\mathrm{vs}\quad H_1: p'<0.3
$$
检验的$p$值为$p=P(x\leqslant 3)$,其中$x\sim b(15,0.3)$,所以
$$
\bm{p}=\sum_{k=0}^{3} \mathrm{C}_{15}^{k} 0.3^{k} 0.7^{15-k} \approx \bm{0.2968679279} > 0.05
$$
\begin{center}
\includegraphics[width=0.3\linewidth]{imgs/2024-06-02-09-31-54.png}
\end{center}
所以不能拒绝原假设,只能认为该人的看法\boldkai{成立}
}
\questionandanswerSolution[4]{
某大学随机调查120名男同学发现有50人非常喜欢看武侠小说而随机调查的85名女同学中有23人喜欢用大样本检验方法在$\alpha=0.05$下确认男女同学在喜爱武侠小说方面有无显著差异?并给出检验的$p$值。
}{
使用大样本$u$检验,
$$
u=\frac{\frac{50}{120}-\frac{23}{85}}{\sqrt{\frac{50}{120}\left( 1-\frac{50}{120} \right) /120 + \frac{23}{85}\left( 1-\frac{23}{85} \right) /85}} \approx 2.21548089304598
$$
$$
\bm{p}=2(1-\Phi(2.21548089304598)) \bm{\approx 0.026728} < 0.05
$$
所以男女同学在喜爱武侠小说方面\boldkai{有显著差异}
}
\questionandanswerSolution[6]{
通常每平方米某种布上的疵点数服从泊松分布现观测该种布100 $\mathrm{m}^{2}$发现有126个疵点在显著性水平为0.05下能否认为该种布每平方米上平均疵点数不超过1个并给出检验的$p$值。
}{
设总体为$X\sim \operatorname{Poi}(\lambda)$,使用大样本检验假设
$$
H_0: \lambda\leqslant 1 \quad\mathrm{vs}\quad H_1: \lambda>1
$$
由于$EX=\operatorname{Var}X = \lambda$,所以$u = \frac{\sqrt{100} \left( \frac{126}{100}-1 \right) }{\sqrt{\frac{126}{100}}} \approx 2.31626409657434$$p$值为
$$
\bm{p} = 1-\Phi\left( 2.31626409657434 \right) \bm{\approx 0.010272} < 0.05
$$
所以拒绝原假设,因此该种布每平方米上平均疵点数\boldkai{超过1个}
}
\questionandanswer[9]{
有—批电子产品共50台产销双方协商同意找出一个检验方案使得当次品率$p\leqslant p_0=0.04$时拒绝的概率不超过0.05,而当$p>p_1=0.30$接受的概率不超过0.10,请你帮助找出适当的检验方案。
}{}
\begin{solution}
{\kaishu
这里的次品率如何定义是指这50台电子产品中次品的频率还是所有生产的产品的频率前者的总体是这50台电子产品并且是不放回抽样那么对应的是超几何分布。后者的总体是所有生产的产品可以近似看作放回抽样那么对应的是二项分布。由于生产的电子产品一般不止50台所以这里认为是后者。
设样本为$x\sim b(n, p)$,由于只有$50$台电子产品用于检验,所以$n\leqslant 50$,而$p$就是次品率。
% 两次检验的假设为
% $$
% H_0:p\leqslant p_0=0.04 \quad\mathrm{vs}\quad H_1:p>0.04
% $$
% $$
% H_0':p>p_1=0.30 \quad\mathrm{vs}\quad H_1':p\leqslant 0.30
% $$
拒绝域为$\{ x>c \}$$P(x, n, p)=\mathrm{C}_{n}^{x} p^{x}(1-p)^{n-x}$。所以需要求出$n$$x$使得
$$
\sum_{x=c+1}^{n} \mathrm{C}_{n}^{x}0.04^{x}(1-0.04)^{n-x} \leqslant 0.05
$$
$$
\sum_{x=0}^{c} \mathrm{C}_{n}^{x} 0.30^{x} (1-0.30)^{n-x} \leqslant 0.10
$$
遍历$n$$c$所有可能的取值($n = 1,2, \cdots ,50$, $c=0,1, \cdots ,n$)即可找到合适的$n$$c$
\begin{minted}[breaklines=true, baselinestretch=1, frame=single, framesep=1em]{python}
from latex2sympy2 import latex2sympy
from sympy.abc import c, n
import pandas as pd
verify1 = latex2sympy(r"\sum_{x=c+1}^{n} \binom{n}{x} 0.04^{x}(1-0.04)^{n-x} \leqslant 0.05")
verify2 = latex2sympy(r"\sum_{x=0}^{c} \binom{n}{x} 0.30^{x} (1-0.30)^{n-x} \leqslant 0.10")
result = []
for _n in range(1, 51):
line = []
for _c in range(0, _n + 1):
line.append(verify1.subs({n:_n, c:_c}) and verify2.subs({n:_n, c:_c}))
for _c in range(_n + 1, 51):
line.append(False)
result.append(line)
pd.DataFrame(result)
\end{minted}
观察结果即可发现在所有结果为 \mintinline{Python}{True} 的位置里,$n$最小取15对应的$c$为2也就是\boldkai{取出15个产品进行检测次品数大于2时就拒绝否则就接受}
}
\end{solution}
\end{enumerate}
\end{document}